2017 AMC 10B Problems/Problem 9

Revision as of 14:40, 16 February 2017 by Apadaki22 (talk | contribs) (Solution)

Problem

A radio program has a quiz consisting of $3$ multiple-choice questions, each with $3$ choices. A contestant wins if he or she gets $2$ or more of the questions right. The contestant answers randomly to each question. What is the probability of winning?

$\textbf{(A)}\ \frac{1}{27}\qquad\textbf{(B)}\ \frac{1}{9}\qquad\textbf{(C)}\ \frac{2}{9}\qquad\textbf{(D)}\ \frac{7}{27}\qquad\textbf{(E)}\ \frac{1}{2}$

Solution

There are two ways that the contestant can win.

Case 1: They guess all three right. This can only happen $\frac1/{3} * \frac1/{3} * \frac1/{3} = \frac1/{27}$ of the time.

Case 2: They guess only two right. We pick one of the questions to get wrong, $3$, and this can happen $\frac1/{3} * \frac1/{3} * \frac2/{3}$ of the time. Thus, $\frac2/{27} * 3$ = $\frac6/{27}$.

So, in total the two cases combined equals $\frac1/{27} + \frac6/{27}$ = $\boxed{\textbf{(D)}\ \frac7/{27]}$ (Error compiling LaTeX. Unknown error_msg).

See Also

2017 AMC 10B (ProblemsAnswer KeyResources)
Preceded by
Problem 8
Followed by
Problem 10
1 2 3 4 5 6 7 8 9 10 11 12 13 14 15 16 17 18 19 20 21 22 23 24 25
All AMC 10 Problems and Solutions

The problems on this page are copyrighted by the Mathematical Association of America's American Mathematics Competitions. AMC logo.png